Question

In: Economics

A firm’s total cost function is given by TC = 3Q^2 + 48. What are the...

A firm’s total cost function is given by TC = 3Q^2 + 48. What are the firm’s fixed cost, variable cost, average fixed cost, average variable cost, and marginal cost functions? Provide a thorough and detailed sketch of the MC, AFC, AVC curves, as well as the average total cost (AC) curve, all in one graph. Find the point where AC is the lowest, show it in the graph.

Solutions

Expert Solution


Related Solutions

Consider a perfectly competitive market where each firm’s total cost function is TC = q^3 –...
Consider a perfectly competitive market where each firm’s total cost function is TC = q^3 – 10q^2 + 50q. a) What is the long run equilibrium price and quantity for each firm? b) The industry demand function is Qd=2000-10p. How many firms are there in the industry in the long run? c) The demand has changed to Qd=4000-18p. Describe the industry’s response to the demand shock and calculate the change in the number of firms in the long run equilibrium.
Suppose the monopolist has a cost function of TC=2q^2+20Q+40, and faces a market demand of P=300-3Q....
Suppose the monopolist has a cost function of TC=2q^2+20Q+40, and faces a market demand of P=300-3Q. a) In order to maximize profits, how much should the firm produce? b) What price should the monopolist charge? c) How much is the consumer surplus in this case? d) How much is the producer surplus?
The total cost function of a perfectly competitive firm is TC= 12+2q^2+4q. What is the firm's...
The total cost function of a perfectly competitive firm is TC= 12+2q^2+4q. What is the firm's optimal quantity at a market price of $12? If over time, the firm could exit and not pay its fixed cost, what would the optimal quantity be?
Suppose that a price-searcher monopolist had a total cost function given by: TC= 20 + 0.5Q...
Suppose that a price-searcher monopolist had a total cost function given by: TC= 20 + 0.5Q +0.2Q2. The demand for the price searcher's product is given by: QD= 100 -20P. Calculate the monopolist's producer surplus.
A firm has the following production function Y=K0.25L0.25. Total cost (TC) is given by TC=wLL+wKK+ZC, where...
A firm has the following production function Y=K0.25L0.25. Total cost (TC) is given by TC=wLL+wKK+ZC, where wLand wK are prices of the two inputs L and K, and ZC are costs that the firm has to pay regardless of production volume as long as it is operative. a)Derive total cost as a function of output C(Y). Derive marginal cost MC and average cost AC. b)Assume that the firm is one of many identical ones operating on a perfectly competitive market...
A firm has the following production function Y=K0.25L0.25. Total cost (TC) is given by TC=wLL+wKK+ZC, where...
A firm has the following production function Y=K0.25L0.25. Total cost (TC) is given by TC=wLL+wKK+ZC, where wL and wK are prices of the two inputs L and K, and ZC are costs that the firm has to pay regardless of production volume as long as it is operative. Derive total cost as a function of output C(Y). Derive marginal cost MC and average cost AC. Assume that the firm is one of many identical ones operating on a perfectly competitive...
Consider a firm with a short run Total Cost (TC) given by TC=2000 + 1000Q -...
Consider a firm with a short run Total Cost (TC) given by TC=2000 + 1000Q - 40Q2 + Q3 . What is the firms marginal cost? What is firm's shut down price?
Consider a firm with the following total cost function: TC = 50 + 6Q + 4Q2 . The marginal cost associated with the given cost function is MC = 6 + 8Q.
Consider a firm with the following total cost function: TC = 50 + 6Q + 4Q2 . The marginal cost associated with the given cost function is MC = 6 + 8Q. Assume the firm is operating in the short-run.A) What are the firm’s fixed costs? What are the firm’s variable costs?B) Calculate average fixed costs, average variable costs, and average total costs.C) Suppose the firm is in a competitive market and is a price taker. Suppose the equilibrium price...
1. The following equation describes a firm’s total cost. TC = 500 + 10Q + Q2...
1. The following equation describes a firm’s total cost. TC = 500 + 10Q + Q2 a. If the firm is a price taker and other firms in the industry sell output at a price of $100, what price should the manager of this firm put on the product? b. What level of output should be produced to maximize profits (or minimize losses)? c. Should the firm keep producing or shut down? Hint: Is P ≥ AVC? d. Calculate profit...
1. Suppose a firm has the following total cost function: TC = 100 + 4q^2 a....
1. Suppose a firm has the following total cost function: TC = 100 + 4q^2 a. What is the minimum price necessary for the firm to earn profit? You must explain your reasoning and process as to how your found the price you found. b. Below what price will the firm shut down in the short run?
ADVERTISEMENT
ADVERTISEMENT
ADVERTISEMENT